Đến nội dung

Hoang Tung 126 nội dung

Có 1000 mục bởi Hoang Tung 126 (Tìm giới hạn từ 26-05-2020)



Sắp theo                Sắp xếp  

#481861 Trận 3 - Phương trình Lượng giác

Đã gửi bởi Hoang Tung 126 on 08-02-2014 - 09:07 trong Thi giải toán Marathon cấp THPT 2014

Ta có:$sin4x=sin(2.2x)=2sin2x.cos2x=2.2sinxcosx.cos2x=4sinx.cosx.cos2x$

          $cos3x+cosx=2.cos\frac{3x+x}{2}.cos\frac{3x-x}{2}=2cos2x.cosx$

Do đó PT $sin4x+2=cos3x+4sinx+cosx< = > 4.sinx.cosx.cos3x+2=2cos2x.cosx+4sinx< = > 4sinx(cosx.cos2x-1)-2(cosx.cos2x-1)=0< = > (2sinx-1)(cosx.cos2x-2)=0< = > 2sinx-1=0$ hoặc $cosx.cos2x-2=0$

-Nếu $2sinx-1=0= > sinx=\frac{1}{2}= > x=\frac{\pi }{6}+k2\pi ,x=\frac{-\pi }{6}+k2\pi$

-Nếu $cosx.cos2x-2=0< = > cosx.cos2x=2< = > cosx(2cos^2x-1)=2< = > 2cos^3x-cosx-2=0$. Đến đây dùng công thức nghiệm bậc 3 để giải là xong

 

Bạn chú ý phần trình bày trước khi nhấn nộp bài nhé!

Phương trình $\sin x=\frac{1}{2}$ bạn giải sai.

Bạn chưa nói rõ $k$ là gì.

Bạn chưa kết luận nghiệm của bài toán.

$\boxed{\text{Điểm bài thi}:2.0}$




#483389 Trận 3 - Hình học

Đã gửi bởi Hoang Tung 126 on 16-02-2014 - 09:06 trong Thi giải toán Marathon cấp THCS 2014

-Kéo dài $AM$ cắt $BC$ tại $K$, $BN$cắt AC tại P,$CP$ cắt AB tại Q.

-Áp dụng tính chất tỉ số diện tích ta có: $\frac{BK}{CK}=\frac{S_{ABK}}{S_{ACK}}=\frac{S_{BMK}}{S_{CMK}}=\frac{S_{ABK}-S_{BMK}}{S_{ACK}-S_{CMK}}=\frac{S_{AMB}}{S_{AMC}}=\frac{S_{AMB}}{S_{AMF}}.\frac{S_{AMF}}{S_{AME}}.\frac{S_{AME}}{S_{AMC}}=\frac{AB}{AF}.\frac{MF}{ME}.\frac{AE}{AC}$(1)

CMTT:$\frac{CP}{PA}=\frac{BC}{BD}.\frac{ND}{NE}.\frac{BE}{BA}$(2)

          $\frac{AQ}{QB}=\frac{AC}{AF}.\frac{PF}{FD}.\frac{DC}{BC}$(3)

Nhân theo vế (1),(2),(3) $= > \frac{BK}{CK}.\frac{CP}{PA}.\frac{AQ}{QB}=(\frac{ME}{MF}.\frac{FN}{ND}.\frac{DP}{PE}).(\frac{AE}{EC}.\frac{DC}{DB}.\frac{BF}{FA})$(4)

 Mặt khác do $AD,BE,CF$ và $DM,EN,PF$ đồng quy nên theo định lý Ceva  ta có:

  $\frac{ME}{MF}.\frac{NF}{ND}.\frac{DP}{PE}=1,\frac{AE}{EC}.\frac{DC}{DB}.\frac{BF}{FA}=1$(5)

Từ (4),(5)$= > \frac{BK}{CK}.\frac{CP}{PA}.\frac{AQ}{QB}=1$

Từ đó theo định lý Ceva đảo $= > AK,BP,CQ$ đồng quy hay $AM,BN,CP$ đồng quy(ĐPCM)

 

Đây là cuộc thi THCS




#478896 Trận 2 - PT, HPT

Đã gửi bởi Hoang Tung 126 on 25-01-2014 - 07:42 trong Thi giải toán Marathon cấp THCS 2014

-Từ phương trình đầu $= > 8x(x-y)-12y(x-y)=0< = > (x-y)(8x-12y)=0< = > (x-y)(2x-3y)=0$

$= > x=y$ hoặc $2x=3y$

-Nếu $x=y$ .Thay vào pt thứ 2 của hệ $= > 4x^2-6x+1=x^2-3x= > 3x^2-3x+1=0< = > 3(x^2-x+\frac{1}{4})+\frac{1}{4}=0< = > 3(x-\frac{1}{2})^2+\frac{1}{4}=0$(Vô lý do $3(x-\frac{1}{2})^2+\frac{1}{4}\geq \frac{1}{4}> 0$)

-Nếu $2x=3y= > y=\frac{2x}{3}$. Thay vào pt thứ 2 của hệ

 $= > 4x^2-6x+1=(\frac{2x}{3})^2-3.\frac{2x}{3}= > 4x^2-6x+1=\frac{4x^2}{9}-2x< = > 4x^2-4x+1=(\frac{2x}{3})^2< = > (2x-1)^2=(\frac{2x}{3})^2= > 2x-1=\frac{2x}{3}$ hoặc $2x-1=\frac{-2x}{3}$ $= > x=\frac{3}{4}$ hoặc $x=\frac{3}{8}$

Từ đó tính được y

___________________
$S = 10$ (châm trước)




#476919 Trận 1 - Số học

Đã gửi bởi Hoang Tung 126 on 12-01-2014 - 16:44 trong Thi giải toán Marathon Chuyên toán 2014

-Với $y=0= > x^2=y^2+\sqrt{y+1}=0+\sqrt{1}=1= > x^2=1= > x=1$(Do $x\geq 0$)

-Với $y> 0= > y+\frac{3}{4}> 0= > y(y+\frac{3}{4})> 0= > 4y^2+3y> 0= > (2y+1)^2> y+1= > 2y+1> \sqrt{y+1}= > y^2+\sqrt{y+1}< y^2+2y+1=(y+1)^2= > x^2< (y+1)^2$(1)

 Do $\sqrt{y+1}> 0= > x^2=y^2+\sqrt{y+1}> y^2= > x^2> y^2$(2)

Từ (1),(2) $= > y^2< x^2< (y+1)^2$ (Điều này luôn vô lý vì giữa 2 số chính phương liên tiếp không có số chính phương nào )

    Vậy nghiệm của phương trình $(x,y)=(1,0)$




#476924 Trận 1 - Số học

Đã gửi bởi Hoang Tung 126 on 12-01-2014 - 17:00 trong Thi giải toán Marathon Chuyên toán 2014

Bài làm : ($MO31$)

-Nếu $y=0= > x^2=y^2+\sqrt{y+1}=0+\sqrt{1}=1= > x^2=1= > x=1$(Do $x\geq 0$)

-Nếu $y> 0= > 4y+3> 0= > y(4y+3)> 0= > 4y^2+4y+1> y+1= > (2y+1)^2> y+1= > 2y+1> \sqrt{y+1}= > x^2=y^2+\sqrt{y+1}< y^2+2y+1=(y+1)^2= > x^2< (y+1)^2$(1)

 Mà $y^2< y^2+\sqrt{y+1}=x^2= > x^2> y^2$(2)

Từ (1) ,(2) $= > y^2< x^2< (y+1)^2$( Điều này vô lý do giữa 2 số chính phương liên tiếp không tồn tại số chính phương nào khác)

  Vậy nghiệm nguyên của phương trình là $(x,y)=(1,0)$

 

$d=10$

$d_{mr}=0;d_{tl}=0;d_{t}=0$

$S=40$




#475191 Trận 1 - PT, HPT

Đã gửi bởi Hoang Tung 126 on 04-01-2014 - 11:21 trong Thi giải toán Marathon cấp THPT 2014

PT $< = > 8x^6+22x^4-x^3+22x^2+8=2\sqrt[3]{6x^{10}+x^9+6x^8}$

Nhận thấy $x=0$ không là nghiệm của phương trình .,

Xét $x$ khác 0 .Chia cả 2 vế của phương trình cho $x^{3}$

PT $< = > 8(x^3+\frac{1}{x^3})+22(x+\frac{1}{x})-1=2\sqrt[3]{6(x+\frac{1}{x})+1}$

$< = > 8(x+\frac{1}{x})\left [ (x+\frac{1}{x})^3-3 \right ]+22(x+\frac{1}{x})-1=2\sqrt[3]{6(x+\frac{1}{x})+1}< = > 8(x+\frac{1}{x})^3-2(x+\frac{1}{x})-1=2\sqrt[3]{6(x+\frac{1}{x})+1}$

Đặt $2(x+\frac{1}{x})=a$ .

PT $< = > a^3-a-1=2\sqrt[3]{3a+1}$

Đặt $\sqrt[3]{3a+1}=t= > t^3-3a-1=0$(1)

Do $a^3-a-1=2\sqrt[3]{3a+1}=2t= > a^3-2t-a-1=0$(2)

Từ (1),(2) $= > t^3-3a-1=a^3-2t-a-1< = > t^3-a^3+2t-2a=0< = > (t-a)(t^2+at+a^2+2)=0< = > t=a$(Do $t^2+at+a^2+2=(t+\frac{a}{2})^2+\frac{3a^2}{4}+2\geq 2> 0$

Do $a=t= > \sqrt[3]{3a+1}=t=a= > a^3-3a-1=0$

 Đến đây ta dùng công thức nghiệm của phương trình bậc 3 rồi giải .

 

 

 

$\boxed{Điểm: 4}$

Thí sinh này chưa đăng kí




#493303 Toppic Các bài toán BĐT qua các kì thi olympic 30/4

Đã gửi bởi Hoang Tung 126 on 16-04-2014 - 16:12 trong Bất đẳng thức - Cực trị

bài 5: 30-4-2010 

cho các số thực a,b,c \geq 1 thỏa mản a+b+c=abc. Chứng minh rằng:

$bc\sqrt{a^{2}-1} +ca\sqrt{b^{2}-1}+ab\sqrt{c^{2}-1}\leq \frac{3\sqrt{3}}{2}abc$

Theo Bunhiacopxki có:$(\sqrt{\frac{c^2-1}{c^2}}+\sqrt{\frac{a^2-1}{a^2}}+\sqrt{\frac{b^2-1}{b^2}})^2\leq 3(3-\frac{1}{a^2}-\frac{1}{b^2}-\frac{1}{c^2})\leq 3(3-\frac{1}{ab}-\frac{1}{bc}-\frac{1}{ac})=3(3-\frac{a+b+c}{abc})=3(3-1)=6= > P\leq \sqrt{6}= > \sum bc\sqrt{a^2-1}\leq \sqrt{6}abc$




#473944 Topic về Bất đẳng thức, cực trị THCS

Đã gửi bởi Hoang Tung 126 on 30-12-2013 - 16:38 trong Bất đẳng thức và cực trị

Cho a,b,c>0 .CMR:

 

$\sum \frac{(b+c-a)^{2}}{(b+c)^{2}+a^{2}}\geq \frac{3}{5}$

Bài này có nhiều cách làm Mình xin nêu ra cách ngắn gọn nhất .

Chuẩn hóa :$a+b+c=3$

BĐT $< = > \sum \frac{a(b+c)}{a^2+(b+c)^2}\leq \frac{6}{5}< = > \sum \frac{a(3-a)}{a^2+(3-a)^2}\leq \frac{6}{5}< = > \sum \frac{1}{2a^2-6a+9}\leq \frac{3}{5}$

Mặt khác ta lại có :$\frac{1}{2a^2-6a+9}\leq \frac{2a+3}{25}< = > a^3+a^3+1\geq 3a^2$(Luôn đúng theo AM-GM 3 số)

$= > \sum \frac{1}{2a^2-6a+9}\leq \frac{2\sum a+9}{25}=\frac{2.3+9}{25}=\frac{3}{5}$(đpcm)

 Dấu = xảy ra khi $a=b=c=1$




#474808 Topic về Bất đẳng thức, cực trị THCS

Đã gửi bởi Hoang Tung 126 on 02-01-2014 - 20:12 trong Bất đẳng thức và cực trị

Chỗ này chuẩn hoá nghĩa là sao???

Có nghĩa là đề bài cho 3 ẩn thì mình có thể giả sử được tổng của 3 ẩn ấy 




#475064 Topic về Bất đẳng thức, cực trị THCS

Đã gửi bởi Hoang Tung 126 on 03-01-2014 - 20:37 trong Bất đẳng thức và cực trị

Vậy giả sử a+b+c=1 cũng được ạ?

Tùy thôi miễn là tổng của 3 số không âm là được




#560382 Topic tổng hợp một số bất đẳng thức trong kì thi MO các nước

Đã gửi bởi Hoang Tung 126 on 19-05-2015 - 19:00 trong Bất đẳng thức - Cực trị

Câu 35 : Cho $a,b,c\in R$ thỏa mãn : $a^{2}+b^{2}+c^{2}=3$ . Chứng minh rằng : 

$\frac{a^{2}}{2+b+c^{2}}+\frac{b^{2}}{2+c+a^{2}}+\frac{c^{2}}{2+a+b^{2}}\geq \frac{(a+b+c)^{2}}{12}$

 

Câu 35: Theo Bunhiacopxki có :

 

   $\frac{a^2}{2+b+c^2}+\frac{b^2}{2+c+a^2}+\frac{c^2}{2+a+b^2}\geq \frac{(a+b+c)^2}{6+(a+b+c)+(a^2+b^2+c^2)}\geq \frac{(a+b+c)^2}{6+\sqrt{3(a^2+b^2+c^2)}+(a^2+b^2+c^2)}=\frac{(a+b+c)^2}{6+\sqrt{3.3}+3}=\frac{(a+b+c)^2}{12}$

 

  Dấu = xảy ra khi  $a=b=c=1$




#564737 Topic tổng hợp một số bất đẳng thức trong kì thi MO các nước

Đã gửi bởi Hoang Tung 126 on 10-06-2015 - 07:15 trong Bất đẳng thức - Cực trị

Spoiler

 

97.(VMO 2006_Bảng B)

 

Tìm số thực $k$ lớn nhất sao cho với mọi $a,b,c$ thực dương thỏa $abc=1$.Ta có BĐT sau:

 

$\frac{1}{a^2}+\frac{1}{b^2}+\frac{1}{c^2}+3k\geq (k+1)(a+b+c)$

 

 

- Chọn $a=x^2,b=c=\frac{1}{x}$

 

BĐT $< = > \frac{1}{x^4}+\frac{1}{\frac{1}{x^2}}+\frac{1}{\frac{1}{x^2}}+3k\geq (k+1)(x^2+\frac{2}{x})$

$< = > \frac{1}{x^4}+2x^2+3k\geq (k+1)(x^2+\frac{2}{x})< = > \frac{1}{x^4}+2x^2-x^2-\frac{2}{x}\geq k(x^2+\frac{2}{x}-3)$

$< = > \frac{1}{x^4}+x^2-\frac{2}{x}\geq k(x^2+\frac{2}{x}-3)< = > \frac{(x-1)^2(x^2+x+1)^2}{x^4}\geq k(\frac{(x-1)^2(x+2)}{x})$

 

- Với $x\not\equiv 1= > \frac{(x^2+x+1)^2}{x^4}\geq \frac{k(x+2)}{x}= > k\leq \frac{(x^2+x+1)^2}{x^3(x+2)}$

 

    $= > k\leq \lim_{x->+\infty }\frac{(x^2+x+1)^2}{x^3(x+2)}=\lim_{x->+\infty }\frac{(1+\frac{1}{x}+\frac{1}{x^2})^2}{1+\frac{2}{x}}=1$  (Do $\lim_{x->+\infty }\frac{1}{x}=0$)

 

  Từ đó $k\leq 1= > > k_{max}=1$. Ta chứng minh $k=1$ là giá trị lớn nhất thỏa mãn bài toán

 

 Thật vậy ,thay $k=1$ vào BĐT 

 

   $= > \frac{1}{a^2}+\frac{1}{b^2}+\frac{1}{c^2}+3\geq 2(a+b+c)< = > b^2c^2+c^2a^2+a^2b^2+3\geq 2(a+b+c)$  (Do $abc=1$)

 

 Theo BĐT Schur bậc 3 ta có :$a^2b^2+b^2c^2+c^2a^2+3=(\sqrt[3]{a^2b^2})^3+(\sqrt[3]{b^2c^2})^3+(\sqrt[3]{c^2a^2})^3+3\sqrt[3]{(a^2b^2.b^2c^2.c^2a^2)}$

$\geq \sqrt[3]{a^2b^2}.\sqrt[3]{b^2c^2}(\sqrt[3]{a^2b^2}+\sqrt[3]{b^2c^2})+\sqrt[3]{b^2c^2}.\sqrt[3]{c^2a^2}(\sqrt[3]{b^2c^2}+\sqrt[3]{c^2a^2})+\sqrt[3]{c^2a^2}.\sqrt[3]{a^2b^2}(\sqrt[3]{c^2a^2}+\sqrt[3]{a^2b^2})$

$\geq \sqrt[3]{(ab^2c)^2}.2\sqrt[3]{ab^2c}+\sqrt[3]{(abc^2)^2}.2\sqrt[3]{abc^2}+\sqrt[3]{(a^2bc)^2}.2\sqrt[3]{a^2bc}$

$=\sqrt[3]{(abc)^3.b^3}+\sqrt[3]{(abc)^3.c^3}+\sqrt[3]{(abc)^3.a^3}=\sqrt[3]{a^3}+\sqrt[3]{b^3}+\sqrt[3]{c^3}=a+b+c$

 

  $= > a^2b^2+b^2c^2+c^2a^2+3\geq 2(a+b+c)$

 

 (Do áp dụng Cosi và $abc=1$)

 

  Do đó ta có ĐPCM. Dấu = xảy ra khi $a=b=c=1$




#564569 Topic tổng hợp một số bất đẳng thức trong kì thi MO các nước

Đã gửi bởi Hoang Tung 126 on 09-06-2015 - 11:22 trong Bất đẳng thức - Cực trị

Bài 93 ( IMO 2012 ) : Cho $a_{2},a_{3},...,a_{n}$ thỏa mãn : $a_{2}.a_{3}...a_{n}=1$ . Chứng minh rằng : 

$(1+a_{2})^{2}(1+a_{3})^{3}...(1+a_{n})^{n}>n^{n}$

Theo Cosi ta có : 

 

  $1+a_{2}\geq 2\sqrt{a_{2}}= > (1+a_{2})^2\geq 2^2.a_{2}$

 $a_{3}+1=a_{3}+\frac{1}{2}+\frac{1}{2}\geq 3\sqrt[3]{\frac{a_{3}}{2^2}}= > (1+a_{3})^3\geq \frac{3^3.a_{3}}{2^2}$

  .....................................................................................................

 $(a_{n}+1)=a_{n}+\frac{1}{n-1}+\frac{1}{n-1}+...+\frac{1}{n-1}\geq n\sqrt[n]{\frac{a_{n}}{(n-1)^{n-1}}}= > (1+a_{n})^n\geq n^{n}.\frac{a_{n}}{(n-1)^{(n-1)}}$

 

 Nhân theo vế các bất đẳng thức 

 

 $= > (1+a_{2})^2(1+a_{3})^3...(1+a_{n})^n\geq 2^{2}.3^{3}...n^{n}.(a_{1}a_{2}...a_{n}).(\frac{1}{2^{2}}.\frac{1}{3^{3}}...\frac{1}{(n-1)^{n-1}})=n^{n}= > (1+a_{2})^2(1+a_{3})^3...(1+a_{n})^{n}\geq n^{n}$

      (Do $a_{2}a_{3}...a_{n}=1$)

 

 Dấu = xảy ra khi $a_{2}=1,a_{3}=\frac{1}{2},...a_{n}=\frac{1}{n-1}= > a_{2}a_{3}...a_{n}=\frac{1}{2.3...(n-1)}$

 

Nhưng điều này vô lý do $a_{2}a_{3}...a_{n}=1$

 

  Do đó dấu = ko xảy ra ,tức là $(a_{2}+1)^2(a_{3}+1)^3...(a_{n}+1)^{n}> n^{n}$




#560384 Topic tổng hợp một số bất đẳng thức trong kì thi MO các nước

Đã gửi bởi Hoang Tung 126 on 19-05-2015 - 19:05 trong Bất đẳng thức - Cực trị

 

Câu 36 : Cho $a,b,c,d\geq 0$ thỏa mãn : $a+b+c+d=4$ .Chứng minh rằng : 

$\frac{a}{a^{3}+8}+\frac{b}{b^{3}+8}+\frac{c}{c^{3}+8}+\frac{d}{d^{3}+8}\leq \frac{4}{9}$

 

 

Câu 36: Ta sẽ chứng minh :

 

  $\frac{a}{a^3+8}\leq \frac{2a+1}{27}$  (1)

 

Thật vậy (1) $< = > (2a+1)(a^3+8)\geq 27a< = > (a-1)^2(2a^2+5a+8)\geq 0$ (Luôn đúng) 

 

 Lập các cái tương tự $= > \sum \frac{a}{a^3+8}\leq \sum \frac{2a+1}{27}=\frac{2\sum a+4}{27}=\frac{2.4+4}{27}=\frac{4}{9}$

 

   Dấu = xảy ra khi $a=b=c=d=1$




#560153 Topic tổng hợp một số bất đẳng thức trong kì thi MO các nước

Đã gửi bởi Hoang Tung 126 on 18-05-2015 - 14:21 trong Bất đẳng thức - Cực trị

 

Câu 30:(Bosnia 2008)

Cho $x,y,z$ là các số thực.CMR

$x^2+y^2+z^2-xy-yz-zx\geq max\left \{ \frac{3}{4}(x-y)^2,\frac{3}{4}(y-z)^2,\frac{3}{4}(z-x)^2 \right \}$

Ta sẽ chứng minh : 

 

  $x^2+y^2+z^2-xy-yz-xz\geq \frac{3}{4}(x-y)^2< = > 4(x^2+y^2+z^2-xy-yz-xz)\geq 3(x-y)^2< = > x^2+y^2+4z^2+2xy-4yz-4xz\geq 0< = > (x+y-2z)^2\geq 0$  (Luôn đúng)

 

Lập luận tương tự $x^2+y^2+z^2-xy-yz-xz\geq \frac{3}{4}(y-z)^2$

 

                                $x^2+y^2+z^2-xy-yz-xz\geq \frac{3}{4}(x-z)^2$

 

Từ đó $= > x^2+y^2+z^2-xy-xz-yz\geq max{\frac{3}{4}(x-y)^2,\frac{3}{4}(y-z)^2,\frac{3}{4}(x-z)^2}$




#564464 Topic tổng hợp một số bất đẳng thức trong kì thi MO các nước

Đã gửi bởi Hoang Tung 126 on 08-06-2015 - 19:43 trong Bất đẳng thức - Cực trị

Bài 89: (Turkish MO 2007)

Cho $a,b,c$ thực dương thỏa mãn: $a+b+c=3$.Chứng minh bất đẳng thức:

 

$\frac{a^2+3b^2}{ab^2(4-ab)}+\frac{b^2+3c^2}{bc^2(4-bc)}+\frac{c^2+3a^2}{ca^2(4-ca)}\geq 4$

Theo Cosi ta có :

 

 $\sum \frac{a^2+3b^2}{ab^2(4-ab)}=\sum \frac{(a^2+b^2)+2b^2}{ab^2(4-ab)}\geq \sum \frac{2ab+2b^2}{ab^2(4-ab)}=\sum \frac{2b(a+b)}{ab^2(4-ab)}$

$=2\sum \frac{a+b}{ab(4-ab)}\geq 2\sum \frac{2\sqrt{ab}}{ab(4-ab)}=4\sum \frac{1}{\sqrt{ab}(4-ab)}$   (1)

 

Mà $8+ab=(4-ab)+(4-ab)+3ab\geq 3\sqrt[3]{3ab(4-ab)^2}= > (8+ab)^3\geq 81ab(4-ab)^2$

$= > ab(4-ab)^2\leq \frac{(8+ab)^3}{81}= > \sqrt{ab}(4-ab)\leq \frac{\sqrt{(8+ab)^3}}9{}= > \frac{1}{\sqrt{ab}(4-ab)}\geq \frac{9}{\sqrt{(8+ab)^3}}$

$= > \sum \frac{1}{\sqrt{ab}(4-ab)}\geq 9\sum \frac{1}{\sqrt{(8+ab)^3}}\geq 9.3\sqrt[3]{\frac{1}{\sqrt{(8+ab)^3(8+bc)^3(8+ac)^3}}}=\frac{27}{\sqrt{(8+ab)(8+bc)(8+ac)}}\geq \frac{27}{\sqrt{\frac{(8+ab+8+bc+8+ac)^3}{27}}}\geq \frac{27}{\sqrt{\frac{(24+\frac{(a+b+c)^2}{3})}{27}}}=\frac{27}{\sqrt{\frac{(25+3)^3}{27}}}=\frac{27}{\sqrt{27^2}}=1$

   $= > \sum \frac{1}{\sqrt{ab}(4-ab)}\geq 1$    (2)

 

   Từ (1) ,(2) $= > \sum \frac{a^2+3b^2}{ab^2(4-ab)}\geq 4$ và ta có ĐPCM

 

Dấu = xảy ra khi $a=b=c=1$




#564607 Topic tổng hợp một số bất đẳng thức trong kì thi MO các nước

Đã gửi bởi Hoang Tung 126 on 09-06-2015 - 15:37 trong Bất đẳng thức - Cực trị

Bài 95 ( USAJMO 2011 ): Cho $a,b,c$ là các số thực dương thỏa mãn : $a^{2}+b^{2}+c^{2}+(a+b+c)^{2}\leq 4$ . Chứng minh rằng : 

$\frac{ab+1}{(a+b)^{2}}+\frac{bc+1}{(b+c)^{2}}+\frac{ca+1}{(c+a)^{2}}\geq 3$

Ta có : $4\geq \sum a^2+(\sum a)^2=2(\sum a^2+\sum ab)= > 2\geq \sum a^2+\sum ab$

 

Từ đó :$\sum \frac{ab+1}{(a+b)^2}=\sum \frac{2ab+2}{2(a+b)^2}\geq \sum \frac{2ab+\sum a^2+\sum ab}{2(a+b)^2}=\sum \frac{(a+b)^2+(c+a)(c+b)}{2(a+b)^2}=\sum (\frac{1}{2}+\frac{(a+c)(c+b)}{2(a+b)^2})=\frac{3}{2}+\frac{1}{2}(\sum \frac{(c+a)(c+b)}{(a+b)^2})\geq \frac{3}{2}+\frac{1}{2}.3\sqrt[3]{\frac{(c+a)(c+b)}{(a+b)^2}.\frac{(b+c)(b+a)}{(c+a)^2}.\frac{(a+b)(a+c)}{(b+c)^2}}=\frac{3}{2}+\frac{3}{2}=3= > \sum \frac{ab+1}{(a+b)^2}\geq 3$

 

  Dấu = xảy ra khi $a=b=c=\frac{1}{\sqrt{3}}$ 




#564748 Topic tổng hợp một số bất đẳng thức trong kì thi MO các nước

Đã gửi bởi Hoang Tung 126 on 10-06-2015 - 08:51 trong Bất đẳng thức - Cực trị

Spoiler

 

98.(Romania 2008)

 

Tìm hằng số $k$ lớn nhất để bất đẳng thức sau đúng:

 

$(a+b+c)(\frac{1}{a+b}+\frac{1}{b+c}+\frac{1}{a+c}-k)\geq k$

Trong đó $a,b,c$ thực dương thỏa $a+b+c=ab+bc+ac$

 

 

BDT $< = > k\leq \frac{(a+b+c)(\frac{1}{a+b}+\frac{1}{b+c}+\frac{1}{c+a})}{1+a+b+c}$

     $= > k\leq max(\frac{(a+b+c)(\frac{1}{a+b}+\frac{1}{b+c}+\frac{1}{c+a})}{1+a+b+c})$ (Do để tìm giá trị lớn nhất của $k$)

 

Ta chứng minh $max(\frac{(a+b+c)(\frac{1}{a+b}+\frac{1}{b+c}+\frac{1}{c+a})}{1+a+b+c})=\frac{9}{8}$

 

   Thật vậy,  BĐT 

 

$< = > 8(a+b+c)(\frac{1}{a+b}+\frac{1}{b+c}+\frac{1}{c+a})\leq 9+\frac{9(a+b+c)^2}{ab+bc+ac}$

  (Do thay $1=\frac{a+b+c}{ab+bc+ac}$)

 

 $< = > 4\left [ (a+b)+(b+c)+(c+a) \right ](\frac{1}{a+b}+\frac{1}{b+c}+\frac{1}{c+a})\leq 9+\frac{9(a+b+c)^2}{ab+bc+ac}$

$< = > 4\left [ \left [ (a+b)+(b+c)+(c+a) \right ](\frac{1}{a+b}+\frac{1}{b+c}+\frac{1}{c+a}) -9\right ]\leq9(\frac{(a+b+c)^2}{ab+bc+ac}-3)$

$< = > 8\left [ \frac{(a-b)^2}{(c+a)(c+b)}+\frac{(b-c)^2}{(b+a)(c+a)}+\frac{(c-a)^2}{(c+b)(a+b)} \right ]\leq\frac{9\left [ (a-b)^2+(b-c)^2+(c-a)^2 \right ]}{ab+bc+ac}$

$< = > (a-b)^2(\frac{9}{ab+bc+ac}-\frac{8}{(a+c)(b+c)})+(b-c)^2(\frac{9}{ab+bc+ac}-\frac{8}{(b+a)(c+a)})+(c-a)^2(\frac{9}{ab+bc+ac}-\frac{8}{(c+b)(a+b)})\geq 0$ (1)

 

 Đặt $S_{a}=\frac{9}{ab+bc+ac}-\frac{8}{(c+a)(b+a)}=\frac{ab+bc+ac+9a^2}{(\sum ab)(c+a)(b+a)}> 0$

       $S_{b}=\frac{9}{ab+bc+ac}-\frac{8}{(b+a)(b+c)}=\frac{ab+bc+ac+9b^2}{(\sum ab)(b+a)(b+c)}> 0$

      $S_{c}=\frac{9}{ab+bc+ac}-\frac{8}{(c+a)(c+b)}=\frac{ab+bc+ac+9c^2}{(\sum ab)(c+a)(c+b)}> 0$

 

Do đó $S_{a}> 0,S_{b}> 0,S_{c}> 0$ nên BĐT (1) đúng và ta có ĐPCM.

 

    Vậy $k_{max}=\frac{9}{8}$ là giá trị cần tìm




#566750 Topic tổng hợp một số bất đẳng thức trong kì thi MO các nước

Đã gửi bởi Hoang Tung 126 on 18-06-2015 - 22:16 trong Bất đẳng thức - Cực trị

Bài 114 (VMO 1999) : Xét các số thực dương $a,b$ sao cho phương trình : 

                                                                                     $ax^{3}-x^{2}+bx-1=0$

có ba nghiệm thực dương ( các nghiệm có thể bằng nhau).

 

Tìm giá trị nhỏ nhất của biểu thức $P=\frac{5a^{2}-3ab+2}{a^{2}(b-a)}$ .

 Gọi $x_{1},x_{2},x_{3}$ là 3 nghiệm dương của phương trình. 

   Theo định lý Viet cho phương trình bậc 3 ta có :

 

  $\left\{\begin{matrix} x_{1}x_{2}+x_{2}x_{3}+x_{3}x_{1}=\frac{b}{a} & & \\ x_{1}+x_{2}+x_{3}=\frac{1}{a} & & \\ x_{1}x_{2}x_{3}=\frac{1}{a} & & \end{matrix}\right.$

 

Ta có :$\frac{b}{a^2}=\frac{b}{a}.\frac{1}{a}=(\sum x_{1}x_{2})(\sum x_{1})$

           $\frac{\sum x_{1}x_{2}}{x_{1}x_{2}x_{3}}=\frac{\frac{b}{a}}{\frac{1}{a}}=b= > \sum \frac{1}{x_{1}}=b= > \frac{b}{a}=(\sum \frac{1}{x_{1}})(\sum x_{1})$

( Do $\sum x_{1}=\frac{1}{a}$)

Theo Cosi ta có :$\frac{1}{a}=x_{1}x_{2}x_{3}=\sum x_{1}\geq 3\sqrt[3]{x_{1}x_{2}x_{3}}= > (x_{1}x_{2}x_{3})^3\geq 27x_{1}x_{2}x_{3}= > x_{1}x_{2}x_{3}\geq \sqrt{27}=3\sqrt{3}$

 

  Từ đó 

$P=\frac{5a^2-3ab+2}{a^2(b-a)}$

$=\frac{\frac{5a^2-3ab+2}{a^3}}{\frac{a^2(b-a)}{a^3}}=\frac{\frac{5}{a}-\frac{3b}{a^2}+\frac{2}{a^3}}{\frac{b}{a}-1}$

$=\frac{5x_{1}x_{2}x_{3}-3(\sum x_{1})(\sum x_{1}x_{2})+2(\sum x_{1})^3}{(\sum x_{1})(\sum \frac{1}{x_{1}})-1}$

$=(x_{1}x_{2}x_{3})(\frac{5x_{1}x_{2}x_{3}-3(\sum x_{1})(\sum x_{1}x_{2})+2(\sum x_{1})^3}{(\sum x_{1})(\sum x_{1}x_{2})-x_{1}x_{2}x_{3}})$

$\geq 3\sqrt{3}.(\frac{5x_{1}x_{2}x_{3}-3\sum x_{1}x_{2}(x_{1}+x_{2})-9x_{1}x_{2}x_{3}+2(\sum x_{1})^3}{\sum x_{1}x_{2}(x_{1}+x_{2})+2x_{1}x_{2}x_{3}})$

$= > P\geq 3\sqrt{3}.\left [ \frac{-4x_{1}x_{2}x_{3}-3\sum x_{1}x_{2}(x_{1}+x_{2})+2(\sum x_{1})^3}{\sum x_{1}x_{2}(x_{1}+x_{2})+2x_{1}x_{2}x_{3}} \right ]$

 

Ta sẽ chứng minh:   $P\geq 12\sqrt{3}$

$< = > \frac{-4x_{1}x_{2}x_{3}-3\sum x_{1}x_{2}(x_{1}+x_{2})+2(\sum x_{1})^3}{\sum x_{1}x_{2}({x_{1}+x_{2}})+2x_{1}x_{2}x_{3}}\geq 4$

$< = > 2(\sum x_{1})^3-3\sum x_{1}x_{2}(x_{1}+x_{2})-4x_{1}x_{2}x_{3}\geq 4\sum x_{1}x_{2}(x_{1}+x_{2})+8x_{1}x_{2}x_{3}$

$< = > 2\sum x_{1}^3+6\sum x_{1}x_{2}(x_{1}+x_{2})+12x_{1}x_{2}x_{3}-3\sum x_{1}x_{2}(x_{1}+x_{2})-4x_{1}x_{2}x_{3}$

$\geq 4\sum x_{1}x_{2}(x_{1}+x_{2})+8x_{1}x_{2}x_{3}$

$< = > 2\sum x_{1}^3\geq \sum x_{1}x_{2}(x_{1}+x_{2})$

$< = > \sum (x_{1}+x_{2})(x_{1}-x_{2})^2\geq 0$

 ( Điều này luôn đúng với $x_{1},x_{2},x_{3}> 0$)

 

  Do đó $P\geq 12\sqrt{3}= > P_{min}=12\sqrt{3}< = > x_{1}=x_{2}=x_{3}< = > a=\frac{1}{3\sqrt{3}},b=\sqrt{3}$




#564828 Topic tổng hợp một số bất đẳng thức trong kì thi MO các nước

Đã gửi bởi Hoang Tung 126 on 10-06-2015 - 17:12 trong Bất đẳng thức - Cực trị

Sửa một chút về điều kiện bài toán là $a,b,c$ không âm sao cho có ít nhất một số dương và $a+b+c=ab+bc+ac$

Nếu thế thì nghe vẻ là bài toán khó hơn đó




#566843 Topic tổng hợp một số bất đẳng thức trong kì thi MO các nước

Đã gửi bởi Hoang Tung 126 on 19-06-2015 - 13:12 trong Bất đẳng thức - Cực trị

Bài 115 (CĐTMO 2006) : Chứng minh rằng với mọi số thực $x,y,z$ thuộc đoạn $[1;2]$ ta luôn có bất đẳng thức :
                                          $(x+y+z)(\frac{1}{x}+\frac{1}{y}+\frac{1}{z})\geq 6(\frac{x}{y+z}+\frac{y}{x+z}+\frac{z}{x+y}).$

                                           Hỏi đẳng thức xảy ra khi và chỉ khi nào ? 

 

  Không mất tổng quát giả sử $x\geq y\geq z$ .Do $2\geq x,y,z\geq 1= > \left\{\begin{matrix} x+y\geq z & & \\ y+z\geq x & & \\ z+x\geq y & & \end{matrix}\right.$

 

BĐT $< = > (\sum x)(\sum \frac{1}{x})\geq 6(\sum \frac{x}{y+z})< = > (\sum x)(\sum \frac{1}{x})-9\geq 6(\sum \frac{x}{y+z}-\frac{3}{2})$

 

 Ta có :$(\sum x)(\sum \frac{1}{x})-9=\sum \frac{(y-z)^2}{yz}$

            $\sum \frac{x}{y+z}-\frac{3}{2}=\sum \frac{(y-z)^2}{2(y+x)(z+x)}$

 

 Do đó BĐT $< = > \sum \frac{(y-z)^2}{yz}\geq 6(\sum \frac{(y-z)^2}{2(y+x)(z+x)})$

$< = > \sum \frac{(y-z)^2}{yz}\geq 3\sum \frac{(y-z)^2}{(y+x)(z+x)}< = > \sum (y-z)^2(\frac{1}{yz}-\frac{3}{(y+x)(z+x)})\geq 0$

$< = > (x-y)^2(\frac{1}{xy}-\frac{3}{(x+z)(y+z)})+(y-z)^2(\frac{1}{yz}-\frac{3}{(y+x)(z+x)})+(x-z)^2(\frac{1}{xz}-\frac{3}{(x+y)(z+y)})\geq 0$

 

  Đặt $\left\{\begin{matrix} S_{a}=\frac{1}{yz}-\frac{3}{(y+x)(z+x)} & & \\ S_{b}=\frac{1}{xz}-\frac{3}{(x+y)(z+y)} & & \\ S_{c}=\frac{1}{xy}-\frac{3}{(x+z)(y+z)} & & \end{matrix}\right.$

 

Ta có :$S_{b}=\frac{1}{xz}-\frac{3}{(x+y)(z+y)}=\frac{xy+yz+y^2-2xz}{xz(x+y)(z+y)}=\frac{x(y-z)+y(y+z)-xz}{xz(x+y)(z+y)}\geq \frac{x(y-z)+xy-xz}{xz(x+y)(z+y)}=\frac{2x(y-z)}{xz(x+y)(z+y)}=\frac{2(y-z)}{z(x+y)(z+y)}\geq 0= > S_{b}\geq 0$

  (Do $y+z\geq x= > y(y+z)\geq xy,y\geq z= > y-z\geq 0$)

          $S_{a}=\frac{1}{yz}-\frac{3}{(x+y)(x+z)}=\frac{x^2+xy+xz-2yz}{yz(x+y)(x+z)}=\frac{x^2+y(x-z)+z(x-y)}{yz(x+y)(x+z)}> 0= > S_{a}> 0$ (Do $x\geq z= > y(x-z)\geq 0,x\geq y= > z(x-y)\geq 0,x^2> 0= > x^2+y(x-z)+z(x-y)> 0$)

  Do đó $S_{a}+S_{b}> 0$

 

Ta có :$S_{b}+S_{c}=\frac{1}{xz}+\frac{1}{xy}-\frac{3}{(x+y)(z+y)}-\frac{3}{(x+z)(y+z)}$

$=\frac{y+z}{xyz}-\frac{3(x+z)+3(x+y)}{(x+y)(y+z)(x+z)}=\frac{y+z}{xyz}-\frac{6x+3y+3z}{(x+y)(y+z)(x+z)}$

  Ta chứng minh $S_{b}+S_{c}\geq 0< = > \frac{y+z}{xyz}\geq \frac{6x+3y+3z}{(x+y)(y+z)(x+z)}< = > (x+y)(x+z)(y+z)^2\geq xyz(6x+3y+3z)< = > (x^2+xz+xy+yz)(y^2+2yz+z^2)\geq 6x^2yz+3xy^2z+3xyz^2< = > x^2y^2+2x^2yz+x^2z^2+xy^2z+2xyz^2+xz^3+xy^3+2xy^2z+xyz^2+y^3z+2y^2z^2+yz^3\geq 6x^2yz+3xy^2z+3xyz^2< = > x^2y^2+x^2z^2+y^3z+yz^3+2y^2z^2+xz^3+xy^3\geq 4x^2yz$

               $< = > yz(y+z)^2+x^2(y^2+z^2)+x(y^3+z^3)\geq 4x^2yz$

  BĐT này đúng do theo Cosi ta có :

   $x^2(y^2+z^2)\geq x^2.2yz=2x^2yz$

   $x(y^3+z^3)\geq x.yz(y+z)\geq x^2yz$ ( Do $y+z\geq x$)

   $yz(y+z)^2\geq x^2yz$ (Do $y+z\geq x$

 

Cộng theo vế $= > yz(y+z)^2+x^2(y^2+z^2)+x(y^3+z^3)\geq 4x^2yz$ (Luôn đúng)

 

   Do đó $S_{b}+S_{c}\geq 0$

 

Từ đó $S_{b}\geq 0,S_{b}+S_{a}> 0,S_{b}+S_{c}\geq 0$ nên theo nguyên lý SOS thì BĐT đề bài được chứng minh. 

 

    Dấu = xảy ra khi $x=y=z$ hoặc $x=2y=2z$ và hoán vị của chúng




#566968 Topic tổng hợp một số bất đẳng thức trong kì thi MO các nước

Đã gửi bởi Hoang Tung 126 on 19-06-2015 - 21:14 trong Bất đẳng thức - Cực trị

Ở đây có được đăng các bài có trong các tạp chí không vậy, nếu như mỗi MO ko thì nghe vẻ hơi ít




#567158 Topic tổng hợp một số bất đẳng thức trong kì thi MO các nước

Đã gửi bởi Hoang Tung 126 on 20-06-2015 - 20:11 trong Bất đẳng thức - Cực trị

Bài 132:(IMO 1984) Cho $a,b,c>0$ thoả mãn $a+b+c=1$.Chứng minh rằng $0\leq ab+bc+ca-2abc\leq \frac{7}{27}$

   Ta có :$\frac{1}{a}+\frac{1}{b}+\frac{1}{c}\geq \frac{9}{a+b+c}=\frac{9}{1}=9$

$= > \frac{ab+bc+ac}{abc}\geq 9abc= > ab+bc+ac-2abc\geq 7abc> 0$

$= > ab+bc+ac-2abc> 0$  (1)

 

  Đồng nhất $a+b+c=1,(a+b+c)^3=1$

 

BDT $ab+bc+ac-2abc\leq \frac{7}{27}< = > (ab+bc+ac)(a+b+c)-2abc\leq \frac{7(a+b+c)^3}{27}$

$< = > 27ab(a+b)+27bc(b+c)+27ac(a+c)+27abc\leq 7(a+b+c)^3$

$< = > 27ab(a+b)+27bc(b+c)+27ac(a+c)+27abc\leq 7(a^3+b^3+c^3)+21ab(a+b)+21bc(b+c)+21ac(a+c)+42abc$

$< = > 7(a^3+b^3+c^3)+15abc\geq 6ab(a+b)+6bc(b+c)+6ac(a+c)$

 

 Theo BDT Schur bậc 3 ta có : $6(a^3+b^3+c^3+3abc)\geq 6(ab(a+b)+bc(b+c)+ac(a+c))$

 

 Theo Cosi ta có : $a^3+b^3+c^3\geq 3abc$

 

Cộng theo vế 2 BDT và rút gọn $= > 7(a^3+b^3+c^3)+15abc\geq 6ab(a+b)+6bc(b+c)+6ac(a+c)$ (2)

 

  Từ (1),(2) $= > 0< ab+bc+ac-2abc\leq \frac{7}{27}$ .Dấu = xảy ra khi $a=b=c=\frac{1}{3}$




#570265 Topic tổng hợp một số bất đẳng thức trong kì thi MO các nước

Đã gửi bởi Hoang Tung 126 on 06-07-2015 - 20:28 trong Bất đẳng thức - Cực trị

 Dạo này TOPIC ít người tham gia vậy ,vẫn những bài toán từ hôm trước




#542705 Topic tập hợp đề thi thử THPT Quốc gia 2014-2015

Đã gửi bởi Hoang Tung 126 on 02-02-2015 - 15:22 trong Thi TS ĐH

Đề thi của chuyên KHTN :

Hình gửi kèm

  • IMG_20150202_1106511.jpg
  • IMG_20150202_1106332.jpg